Bạn chưa đăng nhập. Vui lòng đăng nhập để hỏi bài

Những câu hỏi liên quan
Vương Hoàng Minh
Xem chi tiết
Phan Quang Thái
Xem chi tiết
Hoàng Lê Bảo Ngọc
9 tháng 12 2016 lúc 16:57
Chứng minh \(\sqrt{p}< \sqrt{p-a}+\sqrt{p-b}+\sqrt{p-c}\)

Vì \(\sqrt{p}>0\) nên ta có điều tương đương \(p< \left(\sqrt{p-a}+\sqrt{p-b}+\sqrt{p-c}\right)^2\)

\(\Leftrightarrow p< \left(3p-a-b-c\right)+2\left(\sqrt{p-a}.\sqrt{p-b}+\sqrt{p-b}.\sqrt{p-c}+\sqrt{p-c}.\sqrt{p-a}\right)\)

\(\Leftrightarrow\sqrt{p-a}.\sqrt{p-b}+\sqrt{p-b}.\sqrt{p-c}+\sqrt{p-c}.\sqrt{p-a}>0\) (luôn đúng)

Chứng minh \(\sqrt{p-a}+\sqrt{p-b}+\sqrt{p-c}\le\sqrt{3p}\)

Áp dụng BĐT Bunhiacopxki, ta được : \(\left(1.\sqrt{p-a}+1.\sqrt{p-b}+1.\sqrt{p-c}\right)^2\le3\left(p-a+p-b+p-c\right)\)

\(\Rightarrow\sqrt{p-a}+\sqrt{p-b}+\sqrt{p-c}\le\sqrt{3p}\)

Vậy có đpcm.

thaomi
Xem chi tiết
Kiệt Nguyễn
28 tháng 11 2019 lúc 10:07

Áp dụng BĐT Bu- nhi - a:

\(\sqrt{p-a}+\sqrt{p-b}+\sqrt{p-c}\)\(\le\sqrt{\left(1^2+1^2+1^2\right)\left(p-a+p-b+p-c\right)}\)

\(=\sqrt{3\left(3p-2p\right)}=\sqrt{3p}\)(Vì p là nửa chu vi nên \(a+b+c=2p\))

Khách vãng lai đã xóa
Kiệt Nguyễn
28 tháng 11 2019 lúc 10:08

Dấu "="\(\Leftrightarrow a=b=c\)hay tam giác ABC đều

Khách vãng lai đã xóa
ĂN CỨT CHÓ
28 tháng 11 2019 lúc 11:58

4555555555555555555555555555555555555555555

Khách vãng lai đã xóa
Vương Hoàng Minh
Xem chi tiết
Alice Ngố
Xem chi tiết
Từ Quang MInh
21 tháng 1 2016 lúc 15:09

Bất đẳng thức trên tương đương với 

[(p-a)1/2  +(p-b) 1/2  +(p-c)1/2 ] 2  \(\le\)  3p \(\Leftrightarrow\) p-a+p-b+p-c +2 [ (p-a)1/2(p-b)1/2 + (p-b)1/2(p-c)1/2 + (p-c)1/2(p-a)1/2]\(\le\)3p

 \(\Leftrightarrow\) (p-a)1/2(p-b)1/2 + (p-b)1/2(p-c)1/2 + (p-c)1/2(p-a)1/2\(\le\)p

Theo bất đảng thức cosi thì   (p-a)1/2(p-b)1/2 \(\le\)[(p-a)+(p-b)]/2=c/2; Tương tự (p-b)1/2(p-c)1/2 \(\le\)a/2; (p-c)1/2(p-a)1/\(\le\)b/2; 

Cộng tất cả các vế lại ta được điều phải chứng minh

 

Hoàng Đức
Xem chi tiết
Nguyễn Minh Tuyền
Xem chi tiết
Le Nhat Phuong
30 tháng 8 2017 lúc 17:19

Do a,b,c là 3 cạnh tam giác nên \(a+b-c>0;b+c-a>0;c+a-b>0\)

Đặt \(x=b+c-a>0\)

      \(y=a+c-b>0\)

     \(z=a+b-c>0\)

\(\Rightarrow a=\frac{"y+z"}{2}\)

\(\Rightarrow b=\frac{"x+z"}{2}\)

\(\Rightarrow c=\frac{"x+y"}{2}\)

\(A=\frac{a}{"b+c-a"}+\frac{b}{"a+c-b"}+\frac{c}{"a+b-c"}\)

\(=\frac{"y+z"}{"2x"}+\frac{"x+z"}{"2y"}+\frac{"x+y"}{"2z"}\)

\(=\frac{1}{2}."\frac{x}{y}+\frac{y}{x}+\frac{x}{z}+\frac{z}{x}+\frac{y}{z}+\frac{z}{y}"\)

Áp dụng công thức bdt Cauchy cho 2 số :

\(\frac{x}{y}+\frac{y}{x}\ge2\)

\(\frac{x}{z}+\frac{z}{x}\ge2\)

\(\frac{y}{z}+\frac{z}{y}\ge2\)

Cộng 3 bdt trên, suy ra :

\("\frac{x}{y}+\frac{y}{x}+\frac{x}{z}+\frac{z}{x}+\frac{y}{z}+\frac{z}{y}"\ge6\)

\(\Rightarrow A\ge\frac{1}{2}.6=3\) "dpcm"

P/s: Nhớ thay thế dấu ngoặc kép thành dấu ngoặc đơn nhé

Vo Thi Minh Dao
Xem chi tiết
Akai Haruma
9 tháng 7 2020 lúc 18:50

Lời giải:

Biểu thức không có max mà chỉ có min bạn nhé. Nếu tính min thì làm như sau:

Đặt $x^{10}=a$ với $a\geq 0$

Khi đó: $P=a^{10}-10a+10$

Áp dụng BĐT Cô-si cho các số không âm ta có:

$a^{10}+\underbrace{1+1+1+...+1}_{9}\geq 10\sqrt[10]{a^{10}}=10a$

$\Leftrightarrow a^{10}+9\geq 10a$

$\Rightarrow P=(a^{10}+9)-10a+1\geq 10a-10a+1=1$

Vậy $P_{\min}=1$ khi $a=1\Leftrightarrow x=\pm 1$

Hoàng Phúc
Xem chi tiết
zZz Phan Cả Phát zZz
20 tháng 11 2016 lúc 22:04

Áp dụng định lý Pi-ta-go đó 

Bùi Thị Vân
21 tháng 11 2016 lúc 9:44

\(a,b,c\) là 3 cạnh của tam giác nên \(a,b,c>0\).
Chứng minh bất đẳng thức phụ 
Giả sử: \(\sqrt{2\left(a^2+b^2\right)}\ge a+b\)
            \(\Leftrightarrow2\left(a^2+b^2\right)\ge\left(a+b\right)^2\)
            \(\Leftrightarrow a^2+b^2\ge2ab\) ( luôn đúng)

Giả sử: \(\sqrt{2}\left(a+b+c\right)\le\sqrt{a^2+b^2}+\sqrt{b^2+c^2}+\sqrt{c^2+a^2}\)
             \(\Leftrightarrow2\left(a+b+c\right)\le\sqrt{2\left(a^2+b^2\right)}+\sqrt{2\left(b^2+c^2\right)}+\sqrt{2\left(a^2+c^2\right)}\)
Ta có: \(\sqrt{2\left(a^2+b^2\right)}+\sqrt{2\left(b^2+c^2\right)}+\sqrt{2\left(a^2+c^2\right)}\ge a+b+b+c+a+c\)
        \(\Rightarrow\sqrt{2\left(a^2+b^2\right)}+\sqrt{2\left(b^2+c^2\right)}+\sqrt{2\left(a^2+c^2\right)}\ge2\left(a+b+c\right)\)
Vậy: \(\sqrt{2}\left(a+b+c\right)\le\sqrt{a^2+b^2}+\sqrt{b^2+c^2}+\sqrt{c^2+a^2}\).
Ta chứng minh: \(\sqrt{a^2+b^2}+\sqrt{b^2+c^2}+\sqrt{c^2+a^2}< \sqrt{3}\left(a+b+c\right)\)
Áp dụng bất đẳng thức Bu  - nhi - a  ta có:
\(\left(\sqrt{a^2+b^2}+\sqrt{b^2+c^2}+\sqrt{a^2+c^2}\right)^2\le\left(1+1+1\right)\left(a^2+b^2+b^2+c^2+a^2+c^2\right)\)
                                                                                   \(=6\left(a^2+b^2+c^2\right)\)
Ta cần chứng minh: \(6\left(a^2+b^2+c^2\right)< \left(\sqrt{3}\left(a+b+c\right)\right)^2\)
                     \(\Leftrightarrow2\left(a^2+b^2+c^2\right)< \left(a+b+c\right)^2\)
                     \(\Leftrightarrow a^2+b^2+c^2< 2ab+2bc+2ac\)
                     \(\Leftrightarrow\left(a-b\right)^2+c^2< 2bc+2ac\)(1)
Do \(a,b,c\)là 3 cạnh của tam giác suy ra \(a-b< c\)
Gải sử \(a>b\) suy ra \(\left(a-b\right)^2< c^2\)
Thay vào (1 ) ta có \(c^2+c^2< 2bc+2ac\)
                            \(\Leftrightarrow2c^2< 2c\left(a+b\right)\)
                             \(\Leftrightarrow c< a+b\)( Đúng với a, b, c là 3 cạnh của tam giác)
Vậy BĐT đã được chứng minh.

Nguyễn Thị Thùy Dương
21 tháng 11 2016 lúc 9:52

Binh phuong 3 ve.

=> BDT  trai dc chung minh( de roi)

CM not  BDT phai ( SD  gi nhi?)